Does $sigma$ commute with pullback.












0












$begingroup$



Question. Let $X$ be a set and $T:Xto X$ be a surjective map. Let $mathcal F$ be a collection of subsets of $X$. Is it true that
$$sigma(T^{-1}mathcal F) = T^{-1}(sigma(mathcal F))$$




(Here $sigma(mathcal F)$ denote the $sigma$-algebra generated by $mathcal F$, $T^{-1}(mathcal F)={T^{-1}F:Fin mathcal F}$ etc).



It is clear that the LHS above is contained in the RHS. This is because the RHS is a $sigma$-algebra which contains $T^{-1}mathcal F$.



I am able to prove the reverse containment under the additional assumption that $T$ be bijective.



To do this, let us write $Sigma_{mathcal C}$ to denote the collection of all the $sigma$-algebras on $X$ which contain a given collection of subsets $mathcal C$ of $X$. Then we have a natural map $Sigma_{mathcal C}to Sigma_{T^{-1}mathcal C}$ which takes $mathcal Ain Sigma_{mathcal C}$ to $T^{-1}mathcal A$ and another natural map $Sigma_{T^{-1}mathcal C}to Sigma_{mathcal C}$ which is the inverse of the previous map.
The equivalence follows because $sigma(mathcal F)=bigcap Sigma_{mathcal F}$ and thus $T^{-1}(sigma(mathcal F))= bigcap T^{-1}Sigma_{mathcal F}=bigcapSigma_{T^{-1}mathcal F} = sigma(T^{-1}mathcal F)$.










share|cite|improve this question









$endgroup$

















    0












    $begingroup$



    Question. Let $X$ be a set and $T:Xto X$ be a surjective map. Let $mathcal F$ be a collection of subsets of $X$. Is it true that
    $$sigma(T^{-1}mathcal F) = T^{-1}(sigma(mathcal F))$$




    (Here $sigma(mathcal F)$ denote the $sigma$-algebra generated by $mathcal F$, $T^{-1}(mathcal F)={T^{-1}F:Fin mathcal F}$ etc).



    It is clear that the LHS above is contained in the RHS. This is because the RHS is a $sigma$-algebra which contains $T^{-1}mathcal F$.



    I am able to prove the reverse containment under the additional assumption that $T$ be bijective.



    To do this, let us write $Sigma_{mathcal C}$ to denote the collection of all the $sigma$-algebras on $X$ which contain a given collection of subsets $mathcal C$ of $X$. Then we have a natural map $Sigma_{mathcal C}to Sigma_{T^{-1}mathcal C}$ which takes $mathcal Ain Sigma_{mathcal C}$ to $T^{-1}mathcal A$ and another natural map $Sigma_{T^{-1}mathcal C}to Sigma_{mathcal C}$ which is the inverse of the previous map.
    The equivalence follows because $sigma(mathcal F)=bigcap Sigma_{mathcal F}$ and thus $T^{-1}(sigma(mathcal F))= bigcap T^{-1}Sigma_{mathcal F}=bigcapSigma_{T^{-1}mathcal F} = sigma(T^{-1}mathcal F)$.










    share|cite|improve this question









    $endgroup$















      0












      0








      0





      $begingroup$



      Question. Let $X$ be a set and $T:Xto X$ be a surjective map. Let $mathcal F$ be a collection of subsets of $X$. Is it true that
      $$sigma(T^{-1}mathcal F) = T^{-1}(sigma(mathcal F))$$




      (Here $sigma(mathcal F)$ denote the $sigma$-algebra generated by $mathcal F$, $T^{-1}(mathcal F)={T^{-1}F:Fin mathcal F}$ etc).



      It is clear that the LHS above is contained in the RHS. This is because the RHS is a $sigma$-algebra which contains $T^{-1}mathcal F$.



      I am able to prove the reverse containment under the additional assumption that $T$ be bijective.



      To do this, let us write $Sigma_{mathcal C}$ to denote the collection of all the $sigma$-algebras on $X$ which contain a given collection of subsets $mathcal C$ of $X$. Then we have a natural map $Sigma_{mathcal C}to Sigma_{T^{-1}mathcal C}$ which takes $mathcal Ain Sigma_{mathcal C}$ to $T^{-1}mathcal A$ and another natural map $Sigma_{T^{-1}mathcal C}to Sigma_{mathcal C}$ which is the inverse of the previous map.
      The equivalence follows because $sigma(mathcal F)=bigcap Sigma_{mathcal F}$ and thus $T^{-1}(sigma(mathcal F))= bigcap T^{-1}Sigma_{mathcal F}=bigcapSigma_{T^{-1}mathcal F} = sigma(T^{-1}mathcal F)$.










      share|cite|improve this question









      $endgroup$





      Question. Let $X$ be a set and $T:Xto X$ be a surjective map. Let $mathcal F$ be a collection of subsets of $X$. Is it true that
      $$sigma(T^{-1}mathcal F) = T^{-1}(sigma(mathcal F))$$




      (Here $sigma(mathcal F)$ denote the $sigma$-algebra generated by $mathcal F$, $T^{-1}(mathcal F)={T^{-1}F:Fin mathcal F}$ etc).



      It is clear that the LHS above is contained in the RHS. This is because the RHS is a $sigma$-algebra which contains $T^{-1}mathcal F$.



      I am able to prove the reverse containment under the additional assumption that $T$ be bijective.



      To do this, let us write $Sigma_{mathcal C}$ to denote the collection of all the $sigma$-algebras on $X$ which contain a given collection of subsets $mathcal C$ of $X$. Then we have a natural map $Sigma_{mathcal C}to Sigma_{T^{-1}mathcal C}$ which takes $mathcal Ain Sigma_{mathcal C}$ to $T^{-1}mathcal A$ and another natural map $Sigma_{T^{-1}mathcal C}to Sigma_{mathcal C}$ which is the inverse of the previous map.
      The equivalence follows because $sigma(mathcal F)=bigcap Sigma_{mathcal F}$ and thus $T^{-1}(sigma(mathcal F))= bigcap T^{-1}Sigma_{mathcal F}=bigcapSigma_{T^{-1}mathcal F} = sigma(T^{-1}mathcal F)$.







      measure-theory elementary-set-theory






      share|cite|improve this question













      share|cite|improve this question











      share|cite|improve this question




      share|cite|improve this question










      asked Dec 25 '18 at 7:48









      caffeinemachinecaffeinemachine

      6,63221355




      6,63221355






















          1 Answer
          1






          active

          oldest

          votes


















          2












          $begingroup$

          Consider ${Ain sigma (mathcal F): T^{-1}(A) in sigma (T^{-1} (mathcal F))}$. Verify that this is a sigma algebra which contains $mathcal F$. Hence it contains $sigma (mathcal F)$ which proves the equality without any extra assumptions on $T$.






          share|cite|improve this answer











          $endgroup$













          • $begingroup$
            +1 very nice...
            $endgroup$
            – mathworker21
            Dec 25 '18 at 8:06










          • $begingroup$
            So we don't even need $T$ to be surjective? That is very nice!
            $endgroup$
            – caffeinemachine
            Dec 25 '18 at 8:13










          • $begingroup$
            @caffeinemachine The moral here is: inverse images behave very well w.r.t. set theoretic operations, hence w.r.t. sigma algebras.
            $endgroup$
            – Kavi Rama Murthy
            Dec 25 '18 at 8:18










          • $begingroup$
            @KaviRamaMurthy Yes Prof. Murthy. In fact this is what prompted me to conjecture this in the first place. Somehow I missed considering the set you considered. In hindsight it is natural. I learnt an important lesson from your solution(s). Thank you.
            $endgroup$
            – caffeinemachine
            Dec 25 '18 at 8:29











          Your Answer





          StackExchange.ifUsing("editor", function () {
          return StackExchange.using("mathjaxEditing", function () {
          StackExchange.MarkdownEditor.creationCallbacks.add(function (editor, postfix) {
          StackExchange.mathjaxEditing.prepareWmdForMathJax(editor, postfix, [["$", "$"], ["\\(","\\)"]]);
          });
          });
          }, "mathjax-editing");

          StackExchange.ready(function() {
          var channelOptions = {
          tags: "".split(" "),
          id: "69"
          };
          initTagRenderer("".split(" "), "".split(" "), channelOptions);

          StackExchange.using("externalEditor", function() {
          // Have to fire editor after snippets, if snippets enabled
          if (StackExchange.settings.snippets.snippetsEnabled) {
          StackExchange.using("snippets", function() {
          createEditor();
          });
          }
          else {
          createEditor();
          }
          });

          function createEditor() {
          StackExchange.prepareEditor({
          heartbeatType: 'answer',
          autoActivateHeartbeat: false,
          convertImagesToLinks: true,
          noModals: true,
          showLowRepImageUploadWarning: true,
          reputationToPostImages: 10,
          bindNavPrevention: true,
          postfix: "",
          imageUploader: {
          brandingHtml: "Powered by u003ca class="icon-imgur-white" href="https://imgur.com/"u003eu003c/au003e",
          contentPolicyHtml: "User contributions licensed under u003ca href="https://creativecommons.org/licenses/by-sa/3.0/"u003ecc by-sa 3.0 with attribution requiredu003c/au003e u003ca href="https://stackoverflow.com/legal/content-policy"u003e(content policy)u003c/au003e",
          allowUrls: true
          },
          noCode: true, onDemand: true,
          discardSelector: ".discard-answer"
          ,immediatelyShowMarkdownHelp:true
          });


          }
          });














          draft saved

          draft discarded


















          StackExchange.ready(
          function () {
          StackExchange.openid.initPostLogin('.new-post-login', 'https%3a%2f%2fmath.stackexchange.com%2fquestions%2f3051925%2fdoes-sigma-commute-with-pullback%23new-answer', 'question_page');
          }
          );

          Post as a guest















          Required, but never shown

























          1 Answer
          1






          active

          oldest

          votes








          1 Answer
          1






          active

          oldest

          votes









          active

          oldest

          votes






          active

          oldest

          votes









          2












          $begingroup$

          Consider ${Ain sigma (mathcal F): T^{-1}(A) in sigma (T^{-1} (mathcal F))}$. Verify that this is a sigma algebra which contains $mathcal F$. Hence it contains $sigma (mathcal F)$ which proves the equality without any extra assumptions on $T$.






          share|cite|improve this answer











          $endgroup$













          • $begingroup$
            +1 very nice...
            $endgroup$
            – mathworker21
            Dec 25 '18 at 8:06










          • $begingroup$
            So we don't even need $T$ to be surjective? That is very nice!
            $endgroup$
            – caffeinemachine
            Dec 25 '18 at 8:13










          • $begingroup$
            @caffeinemachine The moral here is: inverse images behave very well w.r.t. set theoretic operations, hence w.r.t. sigma algebras.
            $endgroup$
            – Kavi Rama Murthy
            Dec 25 '18 at 8:18










          • $begingroup$
            @KaviRamaMurthy Yes Prof. Murthy. In fact this is what prompted me to conjecture this in the first place. Somehow I missed considering the set you considered. In hindsight it is natural. I learnt an important lesson from your solution(s). Thank you.
            $endgroup$
            – caffeinemachine
            Dec 25 '18 at 8:29
















          2












          $begingroup$

          Consider ${Ain sigma (mathcal F): T^{-1}(A) in sigma (T^{-1} (mathcal F))}$. Verify that this is a sigma algebra which contains $mathcal F$. Hence it contains $sigma (mathcal F)$ which proves the equality without any extra assumptions on $T$.






          share|cite|improve this answer











          $endgroup$













          • $begingroup$
            +1 very nice...
            $endgroup$
            – mathworker21
            Dec 25 '18 at 8:06










          • $begingroup$
            So we don't even need $T$ to be surjective? That is very nice!
            $endgroup$
            – caffeinemachine
            Dec 25 '18 at 8:13










          • $begingroup$
            @caffeinemachine The moral here is: inverse images behave very well w.r.t. set theoretic operations, hence w.r.t. sigma algebras.
            $endgroup$
            – Kavi Rama Murthy
            Dec 25 '18 at 8:18










          • $begingroup$
            @KaviRamaMurthy Yes Prof. Murthy. In fact this is what prompted me to conjecture this in the first place. Somehow I missed considering the set you considered. In hindsight it is natural. I learnt an important lesson from your solution(s). Thank you.
            $endgroup$
            – caffeinemachine
            Dec 25 '18 at 8:29














          2












          2








          2





          $begingroup$

          Consider ${Ain sigma (mathcal F): T^{-1}(A) in sigma (T^{-1} (mathcal F))}$. Verify that this is a sigma algebra which contains $mathcal F$. Hence it contains $sigma (mathcal F)$ which proves the equality without any extra assumptions on $T$.






          share|cite|improve this answer











          $endgroup$



          Consider ${Ain sigma (mathcal F): T^{-1}(A) in sigma (T^{-1} (mathcal F))}$. Verify that this is a sigma algebra which contains $mathcal F$. Hence it contains $sigma (mathcal F)$ which proves the equality without any extra assumptions on $T$.







          share|cite|improve this answer














          share|cite|improve this answer



          share|cite|improve this answer








          edited Dec 25 '18 at 8:07

























          answered Dec 25 '18 at 7:55









          Kavi Rama MurthyKavi Rama Murthy

          66.9k53067




          66.9k53067












          • $begingroup$
            +1 very nice...
            $endgroup$
            – mathworker21
            Dec 25 '18 at 8:06










          • $begingroup$
            So we don't even need $T$ to be surjective? That is very nice!
            $endgroup$
            – caffeinemachine
            Dec 25 '18 at 8:13










          • $begingroup$
            @caffeinemachine The moral here is: inverse images behave very well w.r.t. set theoretic operations, hence w.r.t. sigma algebras.
            $endgroup$
            – Kavi Rama Murthy
            Dec 25 '18 at 8:18










          • $begingroup$
            @KaviRamaMurthy Yes Prof. Murthy. In fact this is what prompted me to conjecture this in the first place. Somehow I missed considering the set you considered. In hindsight it is natural. I learnt an important lesson from your solution(s). Thank you.
            $endgroup$
            – caffeinemachine
            Dec 25 '18 at 8:29


















          • $begingroup$
            +1 very nice...
            $endgroup$
            – mathworker21
            Dec 25 '18 at 8:06










          • $begingroup$
            So we don't even need $T$ to be surjective? That is very nice!
            $endgroup$
            – caffeinemachine
            Dec 25 '18 at 8:13










          • $begingroup$
            @caffeinemachine The moral here is: inverse images behave very well w.r.t. set theoretic operations, hence w.r.t. sigma algebras.
            $endgroup$
            – Kavi Rama Murthy
            Dec 25 '18 at 8:18










          • $begingroup$
            @KaviRamaMurthy Yes Prof. Murthy. In fact this is what prompted me to conjecture this in the first place. Somehow I missed considering the set you considered. In hindsight it is natural. I learnt an important lesson from your solution(s). Thank you.
            $endgroup$
            – caffeinemachine
            Dec 25 '18 at 8:29
















          $begingroup$
          +1 very nice...
          $endgroup$
          – mathworker21
          Dec 25 '18 at 8:06




          $begingroup$
          +1 very nice...
          $endgroup$
          – mathworker21
          Dec 25 '18 at 8:06












          $begingroup$
          So we don't even need $T$ to be surjective? That is very nice!
          $endgroup$
          – caffeinemachine
          Dec 25 '18 at 8:13




          $begingroup$
          So we don't even need $T$ to be surjective? That is very nice!
          $endgroup$
          – caffeinemachine
          Dec 25 '18 at 8:13












          $begingroup$
          @caffeinemachine The moral here is: inverse images behave very well w.r.t. set theoretic operations, hence w.r.t. sigma algebras.
          $endgroup$
          – Kavi Rama Murthy
          Dec 25 '18 at 8:18




          $begingroup$
          @caffeinemachine The moral here is: inverse images behave very well w.r.t. set theoretic operations, hence w.r.t. sigma algebras.
          $endgroup$
          – Kavi Rama Murthy
          Dec 25 '18 at 8:18












          $begingroup$
          @KaviRamaMurthy Yes Prof. Murthy. In fact this is what prompted me to conjecture this in the first place. Somehow I missed considering the set you considered. In hindsight it is natural. I learnt an important lesson from your solution(s). Thank you.
          $endgroup$
          – caffeinemachine
          Dec 25 '18 at 8:29




          $begingroup$
          @KaviRamaMurthy Yes Prof. Murthy. In fact this is what prompted me to conjecture this in the first place. Somehow I missed considering the set you considered. In hindsight it is natural. I learnt an important lesson from your solution(s). Thank you.
          $endgroup$
          – caffeinemachine
          Dec 25 '18 at 8:29


















          draft saved

          draft discarded




















































          Thanks for contributing an answer to Mathematics Stack Exchange!


          • Please be sure to answer the question. Provide details and share your research!

          But avoid



          • Asking for help, clarification, or responding to other answers.

          • Making statements based on opinion; back them up with references or personal experience.


          Use MathJax to format equations. MathJax reference.


          To learn more, see our tips on writing great answers.




          draft saved


          draft discarded














          StackExchange.ready(
          function () {
          StackExchange.openid.initPostLogin('.new-post-login', 'https%3a%2f%2fmath.stackexchange.com%2fquestions%2f3051925%2fdoes-sigma-commute-with-pullback%23new-answer', 'question_page');
          }
          );

          Post as a guest















          Required, but never shown





















































          Required, but never shown














          Required, but never shown












          Required, but never shown







          Required, but never shown

































          Required, but never shown














          Required, but never shown












          Required, but never shown







          Required, but never shown







          Popular posts from this blog

          How do I know what Microsoft account the skydrive app is syncing to?

          When does type information flow backwards in C++?

          Grease: Live!